Quantcast
  • Register
PhysicsOverflow is a next-generation academic platform for physicists and astronomers, including a community peer review system and a postgraduate-level discussion forum analogous to MathOverflow.

Welcome to PhysicsOverflow! PhysicsOverflow is an open platform for community peer review and graduate-level Physics discussion.

Please help promote PhysicsOverflow ads elsewhere if you like it.

News

PO is now at the Physics Department of Bielefeld University!

New printer friendly PO pages!

Migration to Bielefeld University was successful!

Please vote for this year's PhysicsOverflow ads!

Please do help out in categorising submissions. Submit a paper to PhysicsOverflow!

... see more

Tools for paper authors

Submit paper
Claim Paper Authorship

Tools for SE users

Search User
Reclaim SE Account
Request Account Merger
Nativise imported posts
Claim post (deleted users)
Import SE post

Users whose questions have been imported from Physics Stack Exchange, Theoretical Physics Stack Exchange, or any other Stack Exchange site are kindly requested to reclaim their account and not to register as a new user.

Public \(\beta\) tools

Report a bug with a feature
Request a new functionality
404 page design
Send feedback

Attributions

(propose a free ad)

Site Statistics

205 submissions , 163 unreviewed
5,047 questions , 2,200 unanswered
5,345 answers , 22,709 comments
1,470 users with positive rep
816 active unimported users
More ...

  Rigorous proof of Bohr-Sommerfeld quantization

+ 8 like - 0 dislike
1362 views

Bohr-Sommerfeld quantization provides an approximate recipe for recovering the spectrum of a quantum integrable system. Is there a mathematically rigorous explanation why this recipe works? In particular, I suppose it gives an exact description of the large quantum number asymptotics, which should be a theorem.

Also, is there a way to make the recipe more precise by adding corrections of some sort?


This post has been migrated from (A51.SE)

asked Nov 19, 2011 in Theoretical Physics by Squark (1,725 points) [ revision history ]
recategorized Apr 6, 2014 by dimension10
You know about the WKB approximation, right?

This post has been migrated from (A51.SE)
Yes, it is called WKB and it has its own expansion series (I heard - a rather converging one).

This post has been migrated from (A51.SE)

I have no idea how, but this question magically lost its subcategory... I have added it back now... Strange.  

I would love to find time to read Part V of the ChaosBook.org on this topic to be able to understand how does all this semi-classical quantization business work in 1) larger number of degrees of freedom (quasiperiodic motion) and 2) chaotic motion.

@Void: Please give a link to ''Part V of the ChaosBook.org on this topic'', and ask your question as an independent question in a new thread, as semiclassical approximation is a far wider subject than Bohr-Sommerfled quantization (which works only for integrable systems, where it is exact).

3 Answers

+ 5 like - 0 dislike

Yes, it can be made precise and corresponds to the leading order of the semiclassical expansion (WKB approximation) in $\hbar$. See Faddeev-Yakubovsky's "Lectures on quantum mechanics for mathematics students" (§20, formula (13)). An approach inspired by geometric quantization is explained in chapter 4 in Bates-Weinstein's Lectures on the Geometry of Quantization.

This post has been migrated from (A51.SE)
answered Nov 19, 2011 by Pavel Safronov (1,120 points) [ no revision ]
+ 5 like - 0 dislike

This answer addresses the geometrical origin of the Bohr-Sommerfeld condition. In geometric quantization, the additional structure required beyond the symplectic data of the phase space is a polarization. The quantization spaces are constructed as spaces of polarized sections with respect a polarization. The most "obvious" type of a polarization is the Kahler polarization, where the quantization spaces are spaces of holomorphic sections of a pre-quantum line bundle. Simple examples of systems which can be quantized by means of a Kahler polarization are the Harmonic oscillator and the spin. Another type of polarization is the real polarization (Please see for example the Blau's lecttures), which is locally equivalent to the polarization of a cotangent bundle. A real polarizations foilates the phase space (symplectic manifold) into Lagrangian submanifolds. When the leaves are compact, the quantum Hilbert space consists of sections with support only on certain leaves, which are exactly those which satisfy the Bohr-Sommerfeld condition. In this case, the quantum phase space is generated by distributional sections supported solely on the Bohr-Sommerfeld leaves (This result is due to Snyatycki). For example in the case of the spin, the Bohr-Sommerfeld leaves are small circles at half integer values of the $z$ coordinate in the two dimensional sphere. A more sophisticated example of Bohr-Sommerfeld leaves is the Gelfand-Cetlin system on flag manifolds.

Many classical phase admit both Kahler and real polarizations. It is interesting that in many cases the quantization Hilbert spaces are unitarily equivalent (i.e. the quantization is polarization independent). Please see for example Nohara's exposition.

This post has been migrated from (A51.SE)
answered Nov 20, 2011 by David Bar Moshe (4,355 points) [ no revision ]
+ 3 like - 0 dislike

Contrarily to what is generally believed, a semiclassical approximation is achieved through two different series: One is WKB series and the other is the Wigner-Kirkwood series, the latter being a gradient expansion. In both cases, eigenvalues are obtained by the Bohr-Sommerfeld rule but just at the leading order. I have proved this here (this paper appeared in Proceedings of Royal Society A). This proof is rigorous and quite different from what one finds on standard textbooks. Besides, it produces the full series for the exact eigenvalues with at leading order the ordinary Bohr-Sommerfeld rule.

This post has been migrated from (A51.SE)
answered Nov 20, 2011 by JonLester (345 points) [ no revision ]

Your answer

Please use answers only to (at least partly) answer questions. To comment, discuss, or ask for clarification, leave a comment instead.
To mask links under text, please type your text, highlight it, and click the "link" button. You can then enter your link URL.
Please consult the FAQ for as to how to format your post.
This is the answer box; if you want to write a comment instead, please use the 'add comment' button.
Live preview (may slow down editor)   Preview
Your name to display (optional):
Privacy: Your email address will only be used for sending these notifications.
Anti-spam verification:
If you are a human please identify the position of the character covered by the symbol $\varnothing$ in the following word:
p$\hbar$ysics$\varnothing$verflow
Then drag the red bullet below over the corresponding character of our banner. When you drop it there, the bullet changes to green (on slow internet connections after a few seconds).
Please complete the anti-spam verification




user contributions licensed under cc by-sa 3.0 with attribution required

Your rights
...